Solving the Gaussian Integral for Variance of Gaussian Distribution

Click For Summary
The discussion focuses on solving the Gaussian integral to demonstrate the variance of the Gaussian distribution. Participants emphasize the need to correctly set up the integral, specifically ∫r^2 Exp(-2r^2/2c^2) dr, using integration by parts and substitutions. A key point raised is that the integral of exp(-x^2) over the entire real line is finite and positive, contrary to initial assumptions that it would yield zero. The correct approach involves transforming the integral to the form ∫x^2*exp(-x^2) dx and applying integration techniques. The conversation highlights the importance of understanding the properties of Gaussian integrals in probability theory.
jaobyccdee
Messages
33
Reaction score
0
How to show that the variance of the gaussian distribution using the probability function? I don't know how to solve for ∫r^2 Exp(-2r^2/2c^2) dr .
 
Physics news on Phys.org
Use integration by parts and a substitution. It's really closely related to the integral of Exp(r^2).
 
Last edited:
I tried it. The probability function is 1/(sqrt(2Pi c^2)) * Exp[-r^2/2c] When integrate it from -infinity to infinity, the Exp[r^2] makes everything 0. But we are trying to proof that it's equal to c.
 
jaobyccdee said:
I tried it. The probability function is 1/(sqrt(2Pi c^2)) * Exp[-r^2/2c] When integrate it from -infinity to infinity, the Exp[r^2] makes everything 0. But we are trying to proof that it's equal to c.

Absolutely not: the integral of exp(-x^2) for x going from - infinity to + infinity is a finite, positive value (it is the area under the curve of the graph y = exp(-x^2)); furthermore, this integral can be found everywhere in books and web pages; I will let you find it.

Anyway, you need to find an integral of the form int_{x=-inf..inf} x^2*exp(-x^2) dx, which is obtained from yours by an appropriate change of variables, etc. Integrate by parts, setting u = x and dv = x*exp(-x^2) dx.

RGV
 
thx!:)
 
Question: A clock's minute hand has length 4 and its hour hand has length 3. What is the distance between the tips at the moment when it is increasing most rapidly?(Putnam Exam Question) Answer: Making assumption that both the hands moves at constant angular velocities, the answer is ## \sqrt{7} .## But don't you think this assumption is somewhat doubtful and wrong?

Similar threads

  • · Replies 9 ·
Replies
9
Views
3K
  • · Replies 1 ·
Replies
1
Views
1K
Replies
7
Views
2K
Replies
0
Views
860
Replies
21
Views
2K
Replies
4
Views
2K
  • · Replies 10 ·
Replies
10
Views
2K
Replies
8
Views
2K
  • · Replies 1 ·
Replies
1
Views
2K
  • · Replies 18 ·
Replies
18
Views
2K